Improper integral 1/x^(1/3) from -1 to 8

Click For Summary
The discussion revolves around evaluating the improper integral of 1/x^(1/3) from -1 to 8. The user is uncertain about how to handle the negative values under the cubed root and whether further breakdown of the integral is necessary. It is clarified that cube roots of negative numbers yield negative results, not imaginary ones. The user seeks confirmation on whether the expression 3/2(-1)^(2/3) simplifies to 3/2, which is affirmed. Understanding these concepts is crucial for correctly evaluating the integral.
g-racer
Messages
14
Reaction score
0
Hi there,
I am stuck on this problem: the integral of 1/x^(1/3) from -1 to 8.

I have broken it up into the integral from -1 to 0 and 0 to 8. I am confused as to how the negative values under a cubed root affect things and whether or not I need to break it up further.
I am not sure whether the limit as n goes to -1 of 3/2(x)^(2/3) is real or imaginary. As if -1 is squared first it becomes real but if not then it is imaginary. We haven't done anything on imaginary integrals in class
 
Physics news on Phys.org
g-racer said:
Hi there,
I am stuck on this problem: the integral of 1/x^(1/3) from -1 to 8.

I have broken it up into the integral from -1 to 0 and 0 to 8. I am confused as to how the negative values under a cubed root affect things and whether or not I need to break it up further.
I am not sure whether the limit as n goes to -1 of 3/2(x)^(2/3) is real or imaginary. As if -1 is squared first it becomes real but if not then it is imaginary. We haven't done anything on imaginary integrals in class

Cube roots of negative numbers are negative, not imaginary. E.g. (-2)^(1/3)=-(2)^(1/3).
 
thanks so is 3/2(-1)^2/3 a cubed root squared and therefore =3/2?
 
g-racer said:
thanks so is 3/2(-1)^2/3 a cubed root squared and therefore =3/2?

Yes, it is.
 
Question: A clock's minute hand has length 4 and its hour hand has length 3. What is the distance between the tips at the moment when it is increasing most rapidly?(Putnam Exam Question) Answer: Making assumption that both the hands moves at constant angular velocities, the answer is ## \sqrt{7} .## But don't you think this assumption is somewhat doubtful and wrong?

Similar threads

  • · Replies 7 ·
Replies
7
Views
2K
Replies
2
Views
1K
  • · Replies 105 ·
4
Replies
105
Views
6K
  • · Replies 13 ·
Replies
13
Views
3K
  • · Replies 1 ·
Replies
1
Views
2K
  • · Replies 5 ·
Replies
5
Views
2K
Replies
4
Views
2K
  • · Replies 9 ·
Replies
9
Views
2K
Replies
1
Views
1K
  • · Replies 3 ·
Replies
3
Views
2K